LSAT and Law School Admissions Forum

Get expert LSAT preparation and law school admissions advice from PowerScore Test Preparation.

 ctshoi
  • Posts: 1
  • Joined: Jul 25, 2016
|
#27512
Hi,

On page 3-26, question 8 is a logic game question. For this question, I picked E. I am not understanding why answer C is the correct choice when the rule states "Lalitha performs third only if Norton performs 5th". The question states that "If Norton is scheduled for the fifth performance, which one of the following could be true?".

Based on the question stem, if Norton performs 5th then shouldn't Lalitha perform 3rd?

Answer C, which is the correct answer, states "Lalitha is scheduled for the fourth performance".

Im not understanding why Lalitha should be 4th if the rule states Lalitha should be 3rd if Norton is 5th.
 Clay Cooper
PowerScore Staff
  • PowerScore Staff
  • Posts: 241
  • Joined: Jul 03, 2015
|
#27519
Hi ctshoi,

Thanks for your question.

The problem here is that you have misinterpreted the conditional rule that 'Lalitha performs third only if Norton performs fifth.' Based on the last sentence of your post, I think you have diagrammed that rule as if it said 'Lalitha performs third if Norton performs fifth,' - like this:

N = 5 :arrow: L = 3

But, in fact, there is a big difference between 'only if Norton performs fifth' and 'if Norton performs fifth'; 'if' is a sufficient condition indicator, but 'only if' is a necessary condition indicator. Thus, this rule should actually be diagrammed like this:

L = 3 :arrow: N = 5

I am confident that is where you went wrong. Don't be discouraged though, this is a very common mistake, and now that you know to treat 'only if' differently, you should do better moving forward.

Keep working hard!
User avatar
 raiyanhossain
  • Posts: 3
  • Joined: Jan 28, 2021
|
#83785
I have a question, where did we get the whole set-list for the singers? We know that once L is triggered we get the order for the singers but L as 3 was not triggered. I assumed that N is 5 and P is 2 and M is 1 only when L as 3 is triggered. Any help would be appreciated!
 Rachael Wilkenfeld
PowerScore Staff
  • PowerScore Staff
  • Posts: 1358
  • Joined: Dec 15, 2011
|
#83809
Hi raiyanhossain,

Let's take what we have step by step.

1.KJ (J is immediately after K)
2. M :longline: P (P is sometimes after M)
3. L3 :arrow: N5 (L is 3rd only if N is 5)
4. P2 :arrow: P5 (If P is not 2nd, P is 5th)

That's just a simple rundown of our rules. The local question here tells us that N is in 5th.

I think you have a good instinct that just because N is in 5th, L doesn't have to be in 3rd. But we can make some inferences based on N in 5th. We know if N is 5th, no one else can be. That triggers our fourth rule. If we take the contrapositive of that rule, we see that if P is not in 5th, P must be in 2nd.

So now we know P is 2nd, N is 5th. We also can place M. Since M has to be before P, M has to be in 1. The only other thing we can tell is that KJ have to be in 3/4 or in 6/7 because those are the only two places there is space for the KJ block.

Running through the answer choices we see that answer choice (A) wouldn't work because if J is 6, K would have to be 5th. We know N is 5th here.

Answer choice (B) wouldn't work because P has to be second based on rule 4.

Answer choice (C) is our correct answer---L doesn't have to be in 3, L could be elsewhere.

Answer choice (D) doesn't work because M has to go before P, and that places M in slot 1.

Answer choice (E) doesn't work because M has to go in slot 1 in order to fit before P.

Hope that helps!
 deseraymatteson@gmail.com
  • Posts: 6
  • Joined: Jan 23, 2021
|
#88779
Good morning,

General Question - are the explanations available online in the Testing and Analytics platform or the On Demand Student Center? If so, please let me know and skip the next questions.

#8) Here is my setup for the question:

(New Rule)
_ _ _ _ N _ _

Because N is 5th, L is 3rd (Rule 3)
_ _ L _ N _ _

Because N is 5th, P is must be 2nd (Rule 4)
_ P L _ N _ _

Because P is 2nd, M must be 1st (Rule 2)
M P L _ N _ _

Only place left for K J block is at the end (Rule 1)
M P L _ N K J

Last place for letter O is 4th
M P L O N K J

Please help me understand which rule I am misinterpreting.

Deseray
User avatar
 Stephanie Oswalt
PowerScore Staff
  • PowerScore Staff
  • Posts: 805
  • Joined: Jan 11, 2016
|
#88782
deseraymatteson@gmail.com wrote: Thu Jul 15, 2021 8:31 am Good morning,

General Question - are the explanations available online in the Testing and Analytics platform or the On Demand Student Center? If so, please let me know and skip the next questions.

#8) Here is my setup for the question:

(New Rule)
_ _ _ _ N _ _

Because N is 5th, L is 3rd (Rule 3)
_ _ L _ N _ _

Because N is 5th, P is must be 2nd (Rule 4)
_ P L _ N _ _

Because P is 2nd, M must be 1st (Rule 2)
M P L _ N _ _

Only place left for K J block is at the end (Rule 1)
M P L _ N K J

Last place for letter O is 4th
M P L O N K J

Please help me understand which rule I am misinterpreting.

Deseray
Hi Deseray,

Explanations are available on this forum, and we have a specific sorting system to make it easier for students to find what they're looking for. :-D The forum is sorted by Section Type (LG, LR, RC) -> Test # -> Section (LG Game, LR Section, or RC Passage) -> Question #.

Regarding your question on #10: I have moved it to the appropriate place, so please review the explanation here and the setup here, and let us know if this helps!

Thanks!
User avatar
 Blondeucus
  • Posts: 9
  • Joined: Jan 13, 2023
|
#101791
Adam Tyson wrote: Fri Aug 05, 2016 4:20 pm Hey there J, thanks for the question. That third rule with the conditional relationship between L and N is one that causes a lot of folks confusion, and in many ways it is the key rule of this game. The test authors are counting on a few common mistakes being made.

First, take a look at the indicator words in that rule - "only if". That phrase always indicates a Necessary Condition, which goes to the right of the conditional arrow. Don't confuse "only if" with just plain old "if", which denotes a Sufficient Condition that goes to the left of the arrow.

Since "only if" is connected to N in 5th position, that means N5 is the Necessary Condition. The other condition there, L3, is Sufficient, and goes to the left. That's what gets us the diagram in the explanation here, L3 -> N5.

What you have is a Mistaken Reversal of that rule - you got your Sufficient and Necessary conditions backwards. That's going to make the diagram incorrect and the game a whole lot harder, and the authors expect that. Certain questions will play right into that - take a look at Q8, where they ask what could happen if N is 5th. They want you to reject answer C, thinking that N at 5 forces L to 3, but C is actually the correct answer there - the Mistaken Reversal is thinking that when the Necessary occurs the Sufficient must also occur, when it actually doesn't have to.

As to some of the other not laws, you can probably get away without all the P laws - we know P is 2nd or 5th, so putting not laws at 1, 3, 4, etc. is a bit redundant, although still accurate and maybe helpful to you. The interesting ones here are about M - since M is always before P, and P can never be later than 5th, we can infer that M is never later than 4th, giving us those 2 additional not laws at 5 and 6.

Take another look at the conditional rule, and maybe review conditional reasoning from Lesson 2 (and the common indicator words for both Sufficient and Necessary conditions), and it should become clear. Then, tackle the game again with your new and improved skill set and see what happens!
I've read this comment [Setup and Rule Diagrams thread] and I can understand why that is correct for this game because if this rule isn't true then there is no correct answer for 8 but for the life of me I can't gather that initial understanding just from reading the rule on its own could you give me another example or something to try and lock that in.
 Rachael Wilkenfeld
PowerScore Staff
  • PowerScore Staff
  • Posts: 1358
  • Joined: Dec 15, 2011
|
#101799
Hi Blondeucus,

It's the nature of conditional rules---We can only move one way down the arrow. Let's think about a more familiar conditional relationship.

If you live in Chicago, then you live in the United States.

We'd diagram that as Chicago :arrow: United States

Just like in our L3 :arrow: N5 rule in the game, knowing that someone lives in the United States doesn't mean we know that they live in Chicago. We can't move backward into the arrow. Imagine the arrow as a pokey stick that you don't want to run into. Or like spike strips that puncture your tires if you go the wrong way.

Let's jump in deeper. From the rule Chicago :arrow: United States what do we know and what don't we know?

If you know you are in Chicago, then you know you are in the United States.
If you know you are in the United States, you don't know if you are also in Chicago. United States :arrow: Chicago is a mistaken reversal.
If you know you are not in Chicago, you don't know if you are or are not in the United States. Chicago :arrow: United States is a mistaken negation
If you know you are not in the United States, you know you can't be in Chicago. United States :arrow: Chicago is the contrapositive of our original statement, and is always true.

For number 8, you were looking at the start of a mistaken reversal. It started with knowing the necessary condition (like that you were in the United States), and wanted to test that you recognized that it doesn't prove that the sufficient condition is true.

Hope that helps!

Get the most out of your LSAT Prep Plus subscription.

Analyze and track your performance with our Testing and Analytics Package.